Order of convergence of sequence

Click For Summary
The sequence b_n = (5/6)^(n^2) is being analyzed for its order of convergence, with numerical tests suggesting a value between 1 and 2. However, the discussion indicates that there may be no order of convergence, as applying the definition leads to divergent results depending on the value of p. Specifically, if p is less than or equal to 1, the expression approaches 0, while if p is greater than 1, it approaches infinity. Additionally, a question is raised about whether every sequence possesses an order of convergence, highlighting a potential gap in understanding. The conclusion suggests that this particular sequence may not conform to the standard definition of convergence order.
Damidami
Messages
93
Reaction score
0

Homework Statement



I have to find the order of convergence of the following sequence

b_n = \left( \frac{5}{6} \right)^{n^2}

I have numerically tested that it has to be a real number between 1 and 2, but I can't find it exactly.


I also have this doubt: does every sequence have an order of convergence? (a real number p such that | b_{n+1}/b_{n}^p| converges to a non-zero constant )

Homework Equations





The Attempt at a Solution



Here http://translate.google.com/translate?hl=es&sl=es&tl=en&u=http%3A%2F%2Festudiandoenexactas.wordpress.com%2F2011%2F08%2F29%2Ftp-1-ej-1-3%2F" are my futile attempts at solving it.
 
Last edited by a moderator:
Physics news on Phys.org
It seems to me that there is no order of convergence in this case, according to the definition you gave. Try to plug the formula into the definition of order of convergence.

\frac{5}{6}^{{(n+1)}^2} / {(\frac{5}{6}^{n^2})}^p = \frac{5}{6}^{{(n+1)}^2-pn^2}. If p<=1, then the exponent tends to infinity, so the whole expression tends to 0. If p>1, then the exponent tends to -infinity and the whole expression tends to infinity.
 
Question: A clock's minute hand has length 4 and its hour hand has length 3. What is the distance between the tips at the moment when it is increasing most rapidly?(Putnam Exam Question) Answer: Making assumption that both the hands moves at constant angular velocities, the answer is ## \sqrt{7} .## But don't you think this assumption is somewhat doubtful and wrong?

Similar threads

Replies
4
Views
2K
  • · Replies 2 ·
Replies
2
Views
2K
  • · Replies 5 ·
Replies
5
Views
2K
  • · Replies 1 ·
Replies
1
Views
2K
  • · Replies 7 ·
Replies
7
Views
2K
  • · Replies 1 ·
Replies
1
Views
1K
  • · Replies 34 ·
2
Replies
34
Views
3K
Replies
6
Views
3K
  • · Replies 3 ·
Replies
3
Views
1K
  • · Replies 9 ·
Replies
9
Views
2K